1 Paradojas con el infinito

El infinito es una cosa particularmente extraña: mucha de la intuición que tenemos de nuestra experiencia finita no sirve. Al usar el infinito surgen diferentes paradojas que, como veremos, resultan en comportamientos extravagantes.

1.1 Paradoja de San Petesburgo

Una idea prevalente en teoría económica (y teoría de riesgo) es que el máximo que debería pagarse por un objeto es su ganancia promedio (pensemos, en afán de simplicidad, que lo único que nos importa es obtener más dinero). Por ejemplo, si se sabe que cierta franquicia te regresa en promedio \(\$ 1000\) al año lo más que deberías pagar por una licencia anual serían \(\$ 1000\). ¡Nadie pagaría \(\$ 200,000\) por ganar sólo \(\$ 1000\)! Suena lógico ¿no?.

Si metemos el infinito en la mezcla las cosas ya no son tan lógicas. Para ello te propongo un juego de apuestas. Para éste utilizaremos un mecanismo que genere ceros o unos con igual probabilidad (\(1/2\) es la probabilidad de que salga \(0\) y \(1/2\) de obtener \(1\)). Algunas sugerencias incluyen: arrojar monedas azarosamente y asociar a una cara un \(0\) y a la otra un \(1\), contar cuántas personas vestidas de rojo pasan por un lugar concurrido en un minuto y asociar un \(1\) si es un número par o un \(0\) si es impar, o medir el ruido atmosférico. Para este ejemplo, vamos a considerar el caso en el que arrojamos una moneda tal que ambas caras tienen la misma probabilidad (\(1/2\)) y una de las caras representa un \(1\) mientras que la otra un \(0\).

Las reglas del juego son las siguientes: si en el primer tiro obtienes un \(1\) ganas \(\$2\) y el juego continúa. Si obtienes un \(0\) el juego se detiene y te llevas lo ganado hasta el momento. En los tiros subsecuentes (si no has tenido un \(0\)) las ganancias se duplican siempre que sigas la racha de sacar sólo \(1\)s; es decir, si para el segundo tiro obtienes de nuevo un \(1\) ganas \(\$2\times 2\) (\(\$4\) en total pues duplicaste lo que ya tenías). Si sale \(0\) te detienes. Para el tercer tiro tendrías \(\$4\times 2 = \$8\) si de nuevo obtienes un \(1\). En general, para el tiro \(n\) si no has obtenido ningún \(0\) ganarás \(\$2^n\) (en el tiro \(n\) ganas \(\$2\) pero ya tenías \(\$2^{n-1}\) de los anteriores y los duplicaste). ¿Cuánto pagarías por jugar este juego? (Piénsalo dos segundos: cuál es el precio máximo que tú pagarías por jugar).

Probablemente, si costara \(\$1\) lo jugarías; quizá si costara \(\$2\) también. ¿Lo jugarías si costara \(\$20\)? ¿Si costara \(\$200\) lo harías de todas formas? ¿Y si te costara todo el dinero que posees lo jugarías? La respuesta que te daría la teoría económica “clásica” es que sin importar lo que cueste el juego deberías jugarlo. Veamos por qué.

El objetivo es calcular la ganancia esperada del juego (o la ganancia de un juego “promedio”). Para ello estudiaremos los casos involucrados.

  1. Con probabilidad \(1/2\) ganas \(\$2\) en el primer tiro.
  2. Con probabilidad \(1/2\) duplicas lo que ya tenías en el segundo tiro (duplicaste tu \(\$2\) si obtuviste \(1\)) por lo que en total la probabilidad es \(1/2 \times 1/2 = 1/4\) y las ganancias son \(\$ 2 \times 2 = \$4\)
  3. Con probabilidad \(1/2\) ganas \(\$2\) en el tercer tiro; sin embargo para llegar al tercero tuviste que pasar por los anteriores dos ganando por lo que la probabilidad total de ganar \(\$8\) en el tercer tiro es \(1/2 \times 1/4\).
En general sea \(G_n\) la ganancia esperada haciendo \(n\) tiros entonces tenemos: \[\begin{equation}\nonumber \begin{aligned} G_n & = \underbrace{\frac{1}{2} \times \$2}_{\text{Ganar sólo } 1 \text{tiro}} + \underbrace{\frac{1}{2} \times \frac{1}{2} \times \$2 \times 2}_{\text{Ganar } 2 \text{ tiros}} + \underbrace{\frac{1}{2} \times \frac{1}{2} \times \frac{1}{2} \times \$ 2^2 \times 2)}_{3 \text{ tiros}} \\ & \qquad + \underbrace{\frac{1}{2} \times \frac{1}{2} \times \frac{1}{2} \times \frac{1}{2} \times \$2^3 \times 2)}_{4 \text{ tiros}} + \dots + \underbrace{\overbrace{\frac{1}{2} \times \frac{1}{2} \times \cdots \times \frac{1}{2}}^{n \text{ veces}} \times \$ 2^{n-1} \times 2}_{n \text{ tiros}}\\ & = \sum\limits_{k = 1}^{n} \frac{1}{2^k} \cdot 2^k = \sum\limits_{k=1}^{n} 1. \end{aligned} \end{equation}\] Como, potencialmente podemos tener infinitos tiros (i.e. nada garantiza que la moneda se detiene en algún momento específico, por ejemplo \(n = 500\) o en \(n = 16736\)) entonces la ganancia esperada del juego es: \[\begin{equation} \lim_{n \to \infty} G_n = \lim_{n \to \infty} \sum\limits_{k=1}^{n} 1 = \sum\limits_{k=1}^{\infty} 1 = \infty; \end{equation}\]

es decir, la ganancia esperada del juego es \(\infty\) por lo que, sin importar cuánto te costara, tú deberías pagar ¡porque puedes ganar \(\infty\) y cualquier precio es menor que infinito!

Nota : La paradoja de San Petesburgo radica en que mucha gente acepta que cualquier precio por debajo del precio promedio es buen precio pero, en general, nadie dice que dejaría todos sus bienes por una oportunidad de jugar este juego. ¡Sería ilógico pensar que recuperarías tu inversión!

1.2 Paradoja de Littlewood-Ross

Asume que tienes infinitos billetes de \(\$1\) numerados \(1,2,3, \dots\) y que tienes un banco donde puedes poner todos (de hecho las vas a poner en una hora). El banco cobra una comisión de \(\$1\) por cada transacción usando el billete que tiene más a la mano y sólo permite hacer transacciones de \(10\) en \(10\). En la primer \(1/2\) de hora metes los primeros \(10\) billetes al banco y ellos te cobran usando el billete \(1\) (el que entró primero). Piensas que no importa porque como vas a meter infinitos billetes ¿qué mas da que te quiten el \(1\)? En el siguiente \(1/4\) de hora metes las siguientes \(10\) billetes (i.e. del \(11\) al \(20\)) y el banco cobra su comisión usando el \(2\) (el que entró en segundo lugar). Continúas tu proceso (a pesar de que el \(2\) lo hayan cobrado como comisión) pues como vas a meter infinitos billetes el que el \(2\) no esté no debería importar ¿cierto? El tercer paso ocurre al \(1/8\) de hora cuando metes los \(10\) subsecuentes (\(21\) al \(30\)) y ¡malhora! el banco te cobra la comisión puntualmente usando el billete \(3\) (el que entró en tercer lugar). En general, el paso \(n\) ocurre al \(1/2^n\) de hora cuando metes los billetes \(10(n-1) + 1\) hasta la \(10n\) y el banco te cobra usando el billete \(n\) (el que entró en \(n\)-ésimo lugar). Terminando la hora revisas tu cuenta bancaria y ¡gran sorpresa!…

1.2.1 Tu cuenta no tiene ni un billete

Vamos a modelar el proceso tomando en cuenta la numeración de los billetes. Para ello asignemos a cada billete una variable: \(a_1\) para el billete \(1\), \(a_2\) para el billete \(2\), \(a_3\) para el tercero; \(a_n\) para el \(n\)-ésimo billete. Podemos ver el proceso de ponerlos en el banco como una suma. Por ejemplo para el primer momento tenemos: \[\begin{equation}\nonumber \begin{aligned} B_1 & = \underbrace{a_1 + a_2 + a_3 + a_4 + a_5 + a_6 + a_7 + a_8 + a_9 + a_{10}}_{\text{Metí los primeros } 10 \text{ billetes }} - \overbrace{a_1}^{\text{Comisión usando el billete }1} \\ & = a_2 + a_3 + a_4 + a_5 + a_6 + a_7 + a_8 + a_9 + a_{10} \\ & = \sum\limits_{i = 2}^{10} a_i \end{aligned} \end{equation}\] para el segundo momento tendremos: \[\begin{equation}\nonumber \begin{aligned} B_2 & = \underbrace{a_1 + a_2 + \dots + a_{10} - a_1}_{\text{Lo que ya estaba}} + \underbrace{a_{11} + a_{12} + \dots + a_{20}}_{\text{Los nuevos}} \overbrace{- a_2}^{\text{Comisión}}. \\ & = a_3 + a_4 + a_5 + \dots + a_{20} \\ & = \sum\limits_{i = 3}^{20} a_i \end{aligned} \end{equation}\] En general en el momento \(n\) nuestra cuenta tiene: \[\begin{equation}\nonumber B_n = \sum\limits_{i = n + 1}^{10n} a_i \end{equation}\] Lo que nos gustaría hacer es evaluar la función cuando \(n\) tiende a infinito pero nos topamos con un problema: ¿qué quiere decir la suma de \(\infty\) a \(\infty\)? \[\begin{equation}\nonumber \lim_{n \to \infty} B_n \stackrel{?}{=} \sum\limits_{i = \infty}^{\infty} a_i \stackrel{?}{=} a_{\infty} = \textrm{¿Qué?} \end{equation}\] Para ello un camino es considerar \(f_n(i)\), una función definida como: \[\begin{equation}\nonumber f_n(i) = \begin{cases} 1 & \text{ si } a_i \text{ está en la suma } B_n, \\ 0 & \text{ si no está.} \end{cases} \end{equation}\] En este caso podemos escribir \[\begin{equation}\nonumber B_n' = \sum\limits_{i = 1}^{10n} f_n(i) \end{equation}\] donde los valores de \(B_n'\) y \(B_n\) coinciden para toda \(n\) (-pausa mientras compruebas-). Por como definimos los \(a_i\) tenemos que para cada billete \(i\) éste es cobrado por el banco en el paso \(B_i\) por lo que para cualquier billete \(i\) tenemos que: \[\begin{equation}\nonumber \lim_{n \to \infty} f_n (i) = 0; \end{equation}\]

es decir, eventualmente el billete \(i\) sale como comisión del banco. Como todos los billetes son cobrados como comisión (i.e. salen de la cuenta) concluimos que en el límite, \(\lim_{n \to \infty} B_n\) ¡no hay billetes en el banco!

1.2.2 pero tienes infinito dinero…

Analicemos la suma \(B_n\) definida arriba. Como los \(a_i\) representan que un billete esté en el banco podemos asignar a cada \(a_i\) el valor de \(1\) y modelar los \(B_n\) como la cantidad de dinero en tu cuenta bancaria al momento \(n\). Tenemos entonces que \[\begin{equation} B_n = \sum\limits_{i = n + 1}^{10n} a_i = \sum\limits_{i = n + 1}^{10n} 1 = \underbrace{10n - (n + 1) + 1}_{\# \text{valores entre} n+1 \text{ y } 10n} = 10 n - n = 9n \end{equation}\]

en cuyo caso \(\lim_{n \to \infty} B_n = \lim_{n \to \infty} 9n = \infty\). Por lo que concluimos que en nuestra cuenta de banco hay infinito dinero. ¡Pero no hay un solo billete! (si hubiera billetes ¿cuál es el número de uno de ellos?).

Nota : Como \(B_n = B_n'\) tenemos que en el límite \(B_n'\to \infty\). Aparentemente tenemos una suma sin sumandos que suma a infinito ¡qué locura!

1.2.3 Es más, la cantidad de billetes depende de cómo el banco cobra la comisión

Si el banco en lugar de cobrar la comisión usando el primer billete que metiste tomara el último que metiste (es decir, en la primer transacción el billete \(10\); en la segunda el billete \(20\); en la tercera el \(20\)) tendríamos en este caso que: \[\begin{equation}\nonumber \lim_{n \to \infty} f_n (i) = \begin{cases} 0 \text{ si } i \text{ es múltiplo de } 10, \\ 1 \text{ si } i \text{ no lo es}. \end{cases}; \end{equation}\]

por lo que habría infinitos billetes en el banco (por ejemplo, los billetes marcados con un \(1\) i.e. el \(1, 11, 111, 1111, 11111, \dots\)) están en el banco.

Nota : En toda esta sección las matemáticas son correctas. El problema (y la discusión filosófica) radica en decidir si esta forma de modelar e interpretar el modelo tiene sentido en la vida real. De hecho éste sigue siendo un tema activo en filosofía ¿Qué crees tú?

1.2.4 Explicación de la paradoja

El problema radica en que estamos acostumbrados a intercambiar límites con sumas y pensar que, esencialmente, son lo mismo. ¡No es así! Por un lado tenemos que (en el primer ejemplo) \(\lim_{n \to \infty} f_n (i) = 0\). La intuición nos lleva a pensar que ese límite se “metería” a la suma para acabar con algo del estilo \[\begin{equation}\nonumber B_{\infty}' \stackrel{!}{=} \sum\limits_{i = 1}^{\infty} \lim_{n \to \infty} f_n (i) = \sum\limits_{i = 1}^{\infty} 0 = 0 \end{equation}\] pero en realidad los límites de la suma y la suma de los límites no son lo mismo \[\begin{equation}\nonumber \lim_{n \to \infty} \sum\limits_{i = 1}^{n} \lim_{m \to \infty} f_m (i) \neq \lim_{n \to \infty} \sum\limits_{i = 1}^{n} f_n (i). \end{equation}\]
Advertencia : El límite de una suma es, generalmente distinta, a la suma de los límites. Una forma de recordarlo es: \[\begin{equation}\nonumber \lim \sum\limits^{\infty} \neq \sum\limits^{\infty} \lim \end{equation}\]

1.3 Serie de Grandi (i.e. paradoja del agrupamiento)

Considera la serie \(\mathcal{S} = \sum_{n=0}^{\infty} (-1)^n = 1 - 1 + 1 - 1 + \dots\). Esta serie la podríamos ver (incorrectamente) de dos maneras. La primera es agrupando los \(1\) seguidos de los \(-1\): \[\begin{equation}\nonumber \mathcal{S} = \sum_{n=0}^{\infty} (-1)^n \stackrel{?}{=} (1 - 1) + (1 - 1) + (1 - 1) + (1-1) + \dots = 0 + 0 + 0 + 0 + \dots = 0 \end{equation}\] la otra idea sería dejando un \(1\) libre al inicio y agrupando los \(-1\) seguidos de los \(1\): \[\begin{equation}\nonumber \mathcal{S} = \sum_{n=0}^{\infty} (-1)^n \stackrel{?}{=} 1 + (- 1 + 1) + (- 1 + 1) + (- 1 + 1) + (-1 + 1) + \dots = 1 + 0 + 0 + 0 + \dots = 1 \end{equation}\] Tenemos una serie que, aparentemente, nos da \(1\) y por otro lado nos da \(0\). ¡Esto no tiene ningún sentido! Hay una tercer forma en la que obtenemos \(1/2\): \[\begin{equation}\nonumber \mathcal{S} = \sum_{n=0}^{\infty} (-1)^n \stackrel{?}{=} 1 - (1 - 1 + 1 - 1 + 1 - 1 + 1 - 1 + \dots) = 1 - \mathcal{S} \end{equation}\] de donde despejando \(\mathcal{S}\) tenemos que: \(2 \mathcal{S} = 1\) y por tanto \(\mathcal{S} = \frac{1}{2}\). ¡Las matemáticas no tienen sentido! (-pausa mientras buscas dónde está el error-).
El problema tiene solución: nada nos dice que podemos usar la propiedad asociativa en series infinitas. De hecho las tres formas de derivar un resultado para la serie son incorrectas1: la serie no converge. Este último hecho podemos verlo recordando que la serie en realidad es un límite. Para ello consideramos los primeros \(k\) términos de la serie. \[\begin{equation}\nonumber \sum\limits_{n = 0}^{k} (-1)^n = \begin{cases} 1 & \textrm{ si } k \textrm{ par, }\\ 0 & \textrm{ si } k \textrm{ impar. } \end{cases} \end{equation}\] Si analizamos el límite notaremos que éste no existe: \[\begin{equation} \lim_{k \to \infty} \sum\limits_{n = 0}^{k} (-1)^n = \lim_{k \to \infty} \begin{cases} 1 & \textrm{ si } k \textrm{ par, }\\ 0 & \textrm{ si } k \textrm{ impar. } \end{cases} \equiv \textrm{ No existe. } \end{equation}\] ¿Y dónde estuvo el problema? ¿Por qué concluimos la serie era \(0,1\) ó \(\frac{1}{2}\)? El problema estuvo en asumir que podíamos agrupar términos arbitrariamente. Acabamos de descubrir que en una suma infinita la forma en la que agrupamos los términos sí importa pues en particular: \[\begin{equation} \underbrace{1 + (- 1 + 1) + (- 1 + 1) + (- 1 + 1) + (-1 + 1) + \dots}_{ = 1} \neq \underbrace{(1 - 1) + (1 - 1) + (1 - 1) + (1-1) + \dots}_{= 0}. \end{equation}\]
Advertencia : En general, los sumandos de una serie infinita no son asociativos.

1.4 Paradoja del rearreglo

El problema del rearreglo de series es aún más paradógico. Para ello tomemos un \(\alpha \geq 1\) y consideremos la serie divergente (revisa que es divergente): \[\begin{equation}\nonumber \sum\limits_{i = 1}^{\infty} \frac{1}{2i - 1} = \infty \end{equation}\] Como se va a \(\infty\) la serie entonces existe un término \(m_1 \geq 2\) que cumple que \(m_1\) es el primer término que hace que la suma \(\sum\limits_{i = 1}^{m_1} \frac{1}{2i - 1}\) sea mayor a \(\alpha\). Por ejemplo si \(\alpha = 10\) tenemos que después de \(m_{10} = 68,100,151\) sumandos la suma vale aproximadamente \(10.000000007\) por lo que \(\sum\limits_{i = 1}^{68,100,151} \frac{1}{2i - 1} > \alpha\). Dada la caracterización del \(m_1\) como el primer número que hace que la suma supere a \(\alpha\) tenemos que : \[\begin{equation}\nonumber \alpha \geq \sum\limits_{i = 1}^{m_1 - 1} \frac{1}{2i - 1} \geq \sum\limits_{i = 1}^{m_1} \frac{1}{2i - 1} - \frac{1}{2m_1 - 1} \geq \sum\limits_{i = 1}^{m_1} \frac{1}{2i - 1} - \frac{1}{3} \end{equation}\] donde la última desigualdad se sigue de que \(m_1 > 2\) (-pausa en lo que compruebas que las desigualdades son ciertas-). Sea \(m_2\) el primer entero mayor a \(m_1\) (i.e. \(m_2\) está en el conjunto \(\{m_1 + 1, m_1 + 2, m_1 + 3, \dots \}\)) que cumple que: \[\begin{equation}\nonumber \sum\limits_{i = 1}^{m_1} \frac{1}{2i - 1} - \frac{1}{3} + \sum\limits_{i = m_1 + 1}^{m_2} \frac{1}{2i - 1} > \alpha. \end{equation}\] Donde, aplicando el mismo razonamiento (-pausa mientras verificas que la ecuación de abajo tiene sentido-) de antes tenemos que: \[\begin{equation}\nonumber \sum\limits_{i = 1}^{m_1} \frac{1}{2i - 1} - \frac{1}{3} + \sum\limits_{i = m_1 + 1}^{m_2} \frac{1}{2i - 1} - \frac{1}{9} < \alpha. \end{equation}\]

(-¡No olvides la pausa!-).

Advertencia : En general, los sumandos de una serie infinita no son conmutativos.

1.5 Los 9 alocados

Sabemosque la serie armónica, \(\sum\limits_{n = 1}^{\infty} \frac{1}{n}\), diverge a infinito. No debería sorprendernos entonces que la serie que contiene a todos los enteros positivos con el \(9\) en el denominador diverja: \[\begin{equation}\label{connueve} \mathcal{S} = \frac{1}{9} + \frac{1}{19} + \frac{1}{29} + \dots + \frac{1}{89} + \frac{1}{90} + \frac{1}{91} + \dots + \frac{1}{99} + \frac{1}{109} + \frac{1}{119} + \dots = \infty \end{equation}\] El resultado asombroso (y paradógico) de Kempner (1914) es que la serie que no contiene \(9\) (que, a priori parecería tener términos “más grandes”) converge: \[\begin{equation}\label{sinnueve} \mathcal{Z} = \frac{1}{1} + \frac{1}{2} + \frac{1}{3} + \dots + \frac{1}{8} + \frac{1}{10} + \dots + \frac{1}{18} + \frac{1}{20} + \dots < \infty \end{equation}\] Para ver esto, analizaremos primero la serie que no contiene nueves . Para ello consideremos los sumandos que en el denominador tienen \(k\) dígitos y agrupémoslos. Es decir, fijemos un número máximo de dígitos en el denominador (digamos 2) y agrupemos los términos que tienen dos dígitos. Por ejemplo, para los \(2\) dígitos tenemos: \[\begin{equation}\nonumber \frac{1}{10} + \frac{1}{11} + \frac{1}{12} + \frac{1}{13} + \frac{1}{14} + \frac{1}{15} + \dots + \frac{1}{18} + \frac{1}{20} + \frac{1}{21} + \dots + \frac{1}{86} + \frac{1}{87} + \frac{1}{88} \end{equation}\]

En general todos los números de dos dígitos son de la forma \(d_1 + 10 \times d_2\) donde \(d_1\) son las “unidades” y \(d_2\) las decenas. Por ejemplo \(57\) se puede escribir como \(7 + 10 \times 5\) con \(d_2 = 5\) y \(d_1 = 7\).

Nota : Un ejercicio común en la primaria que es igual a esto es que te hacían escribir un número (digamos el \(762\)) como “7 centenas más 6 decenas más 2 unidades” o bien \(7 \times 100 + 6 \times 10 + 2 \times 1 = 762\).
Si contamos la cantidad de valores que pueden tomar \(d_1\) y \(d_2\) tenemos \(9\) posibilidades para \(d_1\) (\(0,1,2, \dots, 8\)) y sólo \(8\) para \(d_2\) (el \(0\) y el \(9\) no cuentan).En total acabamos con \(8\) veces (porque van de \(1\) a \(8\) las decenas) \(9\) (porque van de \(0\) a \(8\) las unidades). Notamos que cada uno de los términos de dos dígitos son menores o iguales que \(1/10\) por lo que tendríamos: \[\begin{equation}\nonumber \frac{1}{10} + \frac{1}{11} + \frac{1}{12} + \frac{1}{13} + \frac{1}{14} + \frac{1}{15} + \dots + \frac{1}{18} + \frac{1}{20} + \frac{1}{21} + \dots + \frac{1}{86} + \frac{1}{87} + \frac{1}{88} \leq 8 \frac{9}{10} \end{equation}\] Análogamente para el caso de tres dígitos tendremos \(9\) términos por cada unidad \(\times\) \(9\) términos por cada decena y tendremos \(8\) dígitos por cada centena (entre \(100\) y \(800\)) por lo que la suma de los números de tres dígitos es menor que \(8 \frac{9^2}{100}\) donde el \(100\) está ahí porque \(1/100\) es lo mejor que podría pasar (en términos de ser el mayor). Por lo que concluimos: \[\begin{equation}\nonumber \frac{1}{10} + \frac{1}{11} + \frac{1}{12} + \frac{1}{13} + \frac{1}{14} + \frac{1}{15} + \dots + \frac{1}{18} + \frac{1}{20} + \frac{1}{21} + \dots + \frac{1}{86} + \frac{1}{87} + \frac{1}{88} < 8 \frac{9^2}{100} \end{equation}\] Generalizando, para cada grupo de números de \(k\) dígitos notamos que los podemos escribir como \(d_1 \times 10^{(1-1)} + d_2 \times 10^{(2-1)} + d_3 \times 10^{(3-1)} + \dots + d_k \times 10^{(k-1)}\) donde para los primeros \(k-1\) dígitos (\(d_1, d_2, \dots, d_k\)) se pueden tomar \(9\) posibles combinaciones y para el último (el \(d_k\)) sólo \(8\) valores posibles (\(0\) y \(9\) excluidos). Tenemos que la suma de los números de \(k\) dígitos la podemos acotar por \(8 \times \frac{9^{k-1}}{10^{k-1}}\) donde el \(10^{k-1}\) sale de que, \(1/10^{k-1}\) acota a cada uno de los números de \(10^{k}\) dígitos. Por lo que tenemos una cota a la serie : \[\begin{equation}\nonumber 0 \leq \frac{1}{1} + \frac{1}{2} + \frac{1}{3} + \dots + \frac{1}{8} + \frac{1}{10} + \dots + \frac{1}{18} + \frac{1}{20} + \dots \leq \sum\limits_{k = 1}^{\infty} 8 \frac{9^{k-1}}{10^{k-1}} = 8 \underbrace{\sum\limits_{k = 1}^{\infty} \Big( \frac{9}{10} \Big)^{k-1}}_{\text{Geométrica}} = 80. \end{equation}\]

Como la serie es positiva, creciente y acotada entonces converge (no converge a \(80\) de hecho converge a algo \(\leq 80\); en particular es \(\approx 22.920 676 619 264 150 348 16\) Baillie (2008)).

Nota : Intuitivamente la mayor parte de los números de la serie \(\sum_{i=1}^{\infty} \frac{1}{n}\) a largo plazo contienen nueves. Para ver eso, consideremos la probabilidad de que si tomamos un número al azar de \(100\) dígitos éste no contenga ningún \(9\). Para el primer dígito (de izquierda a derecha) hay \(8\) posibilidades, del uno al ocho (-breve pausa mientras corroboras que esto es verdad-), de un total de nueve dígitos posibles, del uno al nueve (-pausa de nuevo-), por lo que la probabilidad de no tener nueves en el primer dígito es \(8/9\); para los siguientes dígitos hay \(9\) posibilidades, del cero al ocho (-pausa para revisar que es cierto-), de un total de \(10\) opciones de dígitos, del cero al nueve, por lo que la probabilidad de no tener nueves en uno de los subsecuentes dígitos es \(9/10\). Tenemos entonces que la probabilidad de no tener nueves en un número de \(100\) dígitos (\(p_{100}\)) es prácticamente cero: \[\begin{equation}\nonumber p_{100} = \underbrace{\frac{8}{9}}_{\textrm{primer dígito}} \times \overbrace{\Big( \frac{9}{10} \Big)^{99}}^{99 \textrm{ dígitos subsecuentes}} \approx 0.0000236. \end{equation}\] En general para un número de \(n\) dígitos (\(n > 1\)) la probabilidad de no tener nueves está dada por: \[\begin{equation}\nonumber p_n = \underbrace{\frac{8}{9}}_{\textrm{primer dígito}} \times \overbrace{\Big( \frac{9}{10} \Big)^{n-1}}^{n-1 \textrm{ dígitos subsecuentes}}, \end{equation}\] donde notamos que a la larga todos tienen \(9\) (i.e. \(\lim_{n \to \infty} p_n = 0\)). Concluimos que (intuitivamente) la serie sin nueves converge porque, a la larga, le estamos quitando prácticamente todos los sumandos.
El que la serie que sólo tiene \(9\) diverge es fácil si usamos que \(\sum_{i = 1}^{n} \frac{1}{n} = \infty\) pues tenemos que: \[\begin{equation}\nonumber \infty = \sum\limits_{i = 1}^{n} \frac{1}{n} = \mathcal{Z} + \mathcal{S} \leq 80 + \mathcal{S} \end{equation}\]

donde \(\mathcal{Z}\) es la suma sin los \(9\) y \(\mathcal{S}\) es la suma sólo con nueves. La única opción para \(\mathcal{S}\) que cumple la desigualdad es que \(\mathcal{S} = \infty\).

1.6 Ejercicios

  1. Lámpara de Thomson Considera una lámpara que tiene un switch de tal forma que sólo está prendida o apagada. Durante un minuto se lleva acabo el siguiente experimento: durante el primer \(1/2\) minuto se prende; durante el siguiente \(1/4\) se apaga; durante el siguiente \(1/8\) se prende de nuevo; en el \(1/16\) consecuente se apaga; durante el \(1/32\) se prende y así sucesivamente. ¿La lámpara estará prendida o apagada al terminar el minuto? Plantea el problema usando una serie, justifica tu planteamiento así como la interpretación del resultado.
  2. Rearreglos 1 Reacomoda las siguientes series para que te den los valores indicados:
    • La serie \(\sum_{i = 1}^{n} \frac{(-1)^{(n+1)}}{n}\) para que converja a \(4\).
    • La serie \(\sum_{i = 1}^{n} (-1)^n\) para que converja a \(652\).
    • La serie \(\sum_{i = 1}^{\infty} \frac{1}{2i - 1}\) para que converja a \(\pi\).
  3. Tu turno de hacer magia Inventa una serie (distinta a las de los ejemplos y ejercicios) que bajo un rearreglo converja a \(2\) y bajo otro rearreglo a \(-1\).
  4. Encuentra el error en el proceso de la siguiente demostración (algo hicieron mal y con ello acabaron con esa conclusión ¿qué hicieron mal?):

    Teorema falso

    $2 = 1$.
    Demostración: Sea $\mathcal{S} = 1 - \frac{1}{2} + \frac{1}{3} - \frac{1}{4} + \frac{1}{5} - \frac{1}{6} + \frac{1}{7} - \frac{1}{8} + \frac{1}{9} - \frac{1}{10} + \frac{1}{11} - \frac{1}{12} + \dots$. Multiplicando la ecuación por $2$ obtenemos que: \begin{equation}\nonumber 2\mathcal{S} = 2 - 1 + \frac{2}{3} - \frac{1}{2} + \frac{2}{5} - \frac{1}{3} + \frac{2}{7} - \frac{1}{4} + \frac{2}{9} - \frac{1}{5} + \frac{2}{11} - \frac{1}{6} \dots \end{equation} Si agrupamos los términos con el mismo denominador tenemos que \begin{equation}\nonumber \begin{aligned} 2\mathcal{S} & = (2 - 1) - \frac{1}{2} + (\frac{2}{3} - \frac{1}{3}) - \frac{1}{4} + (\frac{2}{5} - \frac{1}{5}) - \frac{1}{6} + (\frac{2}{7} - \frac{1}{7}) - \dots \\ & = 1 - \frac{1}{2} + \frac{1}{3} - \frac{1}{4} + \frac{1}{5} - \frac{1}{6} + \frac{1}{7} - \dots \end{aligned} \end{equation} de donde se sigue que: $2\mathcal{S} = \mathcal{S}$ y, dividiendo entre $\mathcal{S}$ se tiene que $2 = 1$.
  5. Determina por qué la siguiente “solución” es incorrecta. Argumenta dónde está el error y por qué es un error. Finalmente, encuentra el verdadero valor al cual converge \(\mathcal{S}\) (no es a \(1\)).

    Ejercicio incorrecto

    Evalúa la convergencia de la serie $\mathcal{S} = \sum\limits_{n = 1}^{\infty} \frac{1}{(2n-1)(2n+1)}$
    Demostración: 1. Sea $\mathcal{S} = \sum\limits_{n = 1}^{\infty} \frac{1}{(2n-1)(2n+1)}$ luego tenemos que: \begin{equation} \begin{aligned} \mathcal{S} & = \frac{1}{3} + \frac{1}{15} + \frac{1}{35} + \frac{1}{63} + \dots \\ & = \underbrace{\Big( 1 - \frac{2}{3} \Big)}_{= 1/3} + \underbrace{\Big(\frac{2}{3} - \frac{3}{5}\Big)}_{= 1/15} + \underbrace{\Big(\frac{3}{5} - \frac{4}{7}\Big)}_{= 1/35} + \dots & = 1 \end{aligned} \end{equation} de donde se sigue que $\mathcal{S} = 1$.
  6. Encuentra el error en el proceso de la siguiente demostración (algo hicieron mal y con ello acabaron con esa conclusión ¿qué hicieron mal?):

    Ejercicio incorrecto

    Evalúa la convergencia de la serie $\mathcal{S} = \sum\limits_{n = 1}^{\infty} \frac{1}{(2n-1)(2n+1)}$
    Demostración: Considera la siguiente serie: \begin{equation}\nonumber \begin{aligned} \mathcal{S} = 1 - 2 + 4 - 8 + 16 - 32 + 64 - \dots = \sum\limits_{n = 1}^{\infty} (-2)^{n-1} \end{aligned} \end{equation} Luego tenemos que: \begin{equation}\nonumber \begin{aligned} \mathcal{S} & = 1 - 2 + 4 - 8 + 16 - 32 + 64 - \dots \\ & = 1 + (-2 + 4) + (-8 + 16) + (-32 + 64) + \dots \\ & = 1 + 2 + 8 + 32 + \dots = \infty; \end{aligned} \end{equation} por otro lado tenemos que: \begin{equation}\nonumber \begin{aligned} \mathcal{S} & = 1 - 2 + 4 - 8 + 16 - 32 + 64 - \dots \\ & = (1 -2) + (4 -8) + (16 - 32) + (64 - 128) + \dots \\ & = -1 -4 -16 - 64 \dots = -\infty \end{aligned} \end{equation} Combinando los resultados tenemos que $\infty = - \infty$.
  7. Crea una demostración falaz (semejante a los ejercicios anteriores) en la que “demuestres” que \(-1 = 1\) usando series de manera incorrecta. Aclara dónde está el error y por qué es falsa la demostración.
  8. Considera un juego como el de la paradoja de San Petesburgo. Asume que en lugar de que en cada turno se dupliquen las ganancias éstas sólo se aumenten \(1.5\) veces. En este caso ¿cuál es la ganancia esperada? ¿si fueras un casino, cuánto es lo mínimo que deberías cobrar a tus clientes por participar en el juego?
  9. Las series son del diablo Lee la siguiente historia (traducida de Hamkins and Lewis (2000)):
    • Modela el proceso de intercambio usando series.
    • Justifica el resultado usando series.

Vamos a suponer que tienes una cantidad infinita de billetes de un dólar numerados [de manera impar] \(1,3,5, \dots\) y que, al entrar a un bar clandestino de mala muerte te encuentras al mismísimo Diablo sentado en una mesa con una pila enorme de dinero. Te sientas y el Diablo te explica que tiene un cariño especial por tus billetes y está listo para pagarte una cantidad importante de dinero por cada uno. Específicamente, está dispuesto a pagar 2 dólares por cada uno de los billetes de un dólar que posees. Para llevar a cabo el intercambio propone una sucesión infinita de transacciones donde en cada una te dará los dos dólares y él te dará uno de sus dólares. La primer transacción tomará \(1/2\) de hora; la segunda \(1/4\); la tercera \(1/8\) y así sucesivamente, de tal forma que después de una hora el intercambio habrá finalizado. El Diablo toma un poco de whisky mientras tú lo piensas: ¿deberías aceptar su propuesta? Quizá piensas que te volverás aún más rico o que, dado que hay un número infinito de billetes, no hará ninguna diferencia. Por lo menos, piensas, no hará daño así que firmas el contrato y comienza el procedimiento. ¿Cómo podría dañarte el trato?

Inicialmente parece que has hecho un buen trato: él te paga dos dólares y tú das sólo uno. El Diablo es muy particular, empero, en el orden en el que los billetes son intercambiados. El contrato estipula que en cada sub-transacción él [¿o ella?] adquiere de ti tu billete con número más bajo y te paga con dos billetes con número más alto. Entonces, en tu primer transacción él acepta tu billete número \(1\) y te paga con billetes numerados \(2\) y \(4\). En la siguiente transacción él te compra el billete número \(2\) (que recién te pagó) y te da billetes numerados \(6\) y \(8\). En el siguiente turno te compra el billete número \(3\) y te da billetes \(10\) y \(12\) y así. Cuando los intercambios han terminado ¿qué descubres? ¡No tienes nada de billetes!

2 Viendo al futuro

Aunque las series infinitas son divertidas no lo son todo en esta vida. En particular desde hace mucho tiempo los matemáticos han dejado de investigar las series infinitas en los reales por sí mismas. Hay varias áreas en matemáticas que estudian series: por ejemplo, el estudio de funciones analíticas, de series de Fourier o (en general) bases ortonormales (como las de álgebra matricial). Una de las generalizaciones que me parecen más interesantes son las caminatas aleatorias.

Imagina una persona que está centrada en el \(0\). Asume que la persona tiene una moneda con caras águila y sol que tira de manera aleatoria según los resultados decide su próximo movimiento: si obtiene águila se mueve a la derecha (\(+1\)) mientras que si obtiene un sol se mueve a la izquierda (\(-1\)). Considera que este proceso continúa indefinidamente. La posición de la persona al tiro \(n\), \(\mathcal{P}_n\), está dada por: \[\begin{equation} \mathcal{P}_n = X_1 + X_2 + \dots + X_n = \sum\limits_{i = 1}^{n} X_i. \end{equation}\] donde las variables aleatorias \(X_k\) (\(k = 1,2,\dots, n\)) representan el resultado asignado al \(k\)-ésimo tiro de la moneda (\(+1 \to \text{Águila}\) y \(-1 \to \text{Sol}\)). Por poner un ejemplo si una persona ha hecho tres tiros y obtuvo Águila, Sol, Sol tendrá que: \[\begin{equation} \mathcal{P}_n = \underbrace{1}_{X_1} + \underbrace{-1}_{X_2} + \underbrace{-1}_{X_3}; \end{equation}\] y su posición será el \(-1\) al tiro \(3\). Una persona que obtuvo en tres tiros Águila, Águila, Águila tendrá: \[\begin{equation} \mathcal{P}_n = \underbrace{1}_{X_1} + \underbrace{1}_{X_2} + \underbrace{1}_{X_3} \end{equation}\]

y su posición al tiro \(3\) será \(3\).

Modelos como éste pueden parecer completamente ociosos; empero, son bastante útiles en la práctica. La hipótesis de la caminata aleatoria del mercado financiero establece que los precios de una acción se mueven de manera aleatoria aumentando y disminuyendo cada segundo sin dependencia en el pasado. De manera simple, la idea es que el precio de una acción al segundo \(n\) (llamémosla \(\mathcal{P}_n\)) aumenta o disminuye \(\pm 1\) aleatoriamente con distintas probabilidades (por ejemplo puede aumentar con probabilidad \(0.64\) y disminuir con probabilidad \(0.36\)). Es decir, el precio de una acción puede escribirse como una caminata aleatoria: \[\begin{equation} \mathcal{P}_n = \sum\limits_{i = 1}^{n} X_i \end{equation}\]

donde las \(X_i\) son variables aleatorias que toman valores \(\pm 1\) con distintas probabilidades.

Hay varias preguntas interesantes en este tema, por ejemplo ¿cuál es la probabilidad de que el precio de una acción llegue a \(0\) (ruina)?, ¿cuándo es el momento óptimo para vender una acción dado que su precio fluctúa aleatoriamente?, dados \(\$1000\) ¿cuánto de cada acción comprar si quiero obtener un beneficio a un año de al menos \(\$100\) con \(99\%\) de probabilidad? Dar respuestas a estos cuestionamientos así como encontrar modelos más adecuados requiere de herramientas teóricas matemáticas pesadas (muchas basadas en los temas de Análisis Matemático) por lo que dar una referencia es complicado (uno de los clásicos son los dos tomos de Shreve); lo que puedo sugerir, si esto te interesa es echarle un ojo A Random Walk Down Wall Street pues es un libro de divulgación.

3 Referencias

Baillie, Robert. 2008. “Summing the Curious Series of Kempner and Irwin.”

Candelpergher, Bernard. 2017. Ramanujan Summation of Divergent Series. Vol. 2185. Springer.

Hamkins, Joel David, and Andy Lewis. 2000. “Infinite Time Turing Machines.” The Journal of Symbolic Logic 65 (2). Cambridge University Press: 567–604.

Hardy, Godfrey Harold. 2000. Divergent Series. Vol. 334. American Mathematical Soc.

Kempner, AJ. 1914. “A Curious Convergent Series.” The American Mathematical Monthly 21 (2). JSTOR: 48–50.


  1. Técnicamente ninguna forma es incorrecta e incluso hay gente que se ha dedicado a estudiar cómo sacar los límites de las series cuando, por ejemplo queremos que esa serie sea \(1\) o que sea \(-1\) y analiza qué implica para el resto de las sumas. Puedes ver, por ejemplo a Hardy (2000) y a Candelpergher (2017).

Contáctame en rzepeda17@gmail.com